Medical researcher

This topic has expert replies
User avatar
Senior | Next Rank: 100 Posts
Posts: 76
Joined: Mon Jan 20, 2014 3:24 am
Thanked: 3 times

Medical researcher

by Amadalia » Sat Apr 19, 2014 6:03 am
Medical researcher: Asexpected, records covering
the last four years of ten major hospitals
indicate that babies born prematurely were
more likely to have low birth weights and to
suffer from health problems than were babies
not born prematurely. These records also
indicate that mothers who had received
adequate prenatal care were less likely to have
low birth weight babies than were mothers who
had received inadequate prenatal care.
Adequate prenatal care, therefore, significantly
decreases the risk of low birth weight babies.
Which one of the following, if true, most weakens the
medical researcher's argument?
(A) The hospital records indicate that many babies
that are born with normal birth weights are
born to mothers who had inadequate prenatal
care.
(B) Mothers giving birth prematurely are routinely
classified by hospitals as having received
inadequate prenatal care when the record of
that care is not available.
(C) The hospital records indicate that low birth
weight babies were routinely classified as
having been born prematurely.
(D) Some babies not born prematurely, whose
mothers received adequate prenatal care,
have low birth weights.
(E) Women who receive adequate prenatal care are
less likely to give birth prematurely than are
women who do not receive adequate prenatal
care.

the OA is B [spoiler]but for me


both B and C are correct and weakens the arguments

the premises are as follow
indadequate prenatal care------->premature birth
premature Birth ------->low weighs

Conclusion
inadequate prenatal care----->low weigh

B weakens the conculsion from the first side
C weakens the conclusion from the second side
[/spoiler]
Please any hint would be highly appreciated!!!

Thanks

User avatar
Senior | Next Rank: 100 Posts
Posts: 76
Joined: Mon Jan 20, 2014 3:24 am
Thanked: 3 times

by Amadalia » Sat Apr 19, 2014 6:21 am
Sorry ,
I was wrong about the premises
The Answer B is clearly the one!!!

thanks!!!

Junior | Next Rank: 30 Posts
Posts: 17
Joined: Fri Feb 14, 2014 10:05 am
Thanked: 2 times

by ajaysingh24 » Sun Apr 20, 2014 5:14 am
Amadalia wrote:Sorry ,
I was wrong about the premises
The Answer B is clearly the one!!!

thanks!!!
what is wrong with the premises you have given ? .. it seemed right to me

User avatar
Senior | Next Rank: 100 Posts
Posts: 76
Joined: Mon Jan 20, 2014 3:24 am
Thanked: 3 times

by Amadalia » Sun Apr 20, 2014 5:29 am
Look

I've said

indadequate prenatal care------->premature birth
premature Birth ------->low weighs

I was wrong about the bolded premise, the stimulus said
mothers who had received adequate prenatal care were less likely to have low birth weight
means inadequate prenatal care----->low birth weigh and NOT premature birth

Hope it's clear now, please tell me if you need more explanation!!!
regards

Newbie | Next Rank: 10 Posts
Posts: 3
Joined: Sun Jun 09, 2013 1:27 am

by mayankgupta1912 » Sun Apr 20, 2014 3:05 pm
Please check the argument. The second and the third sentence are saying the same things.

User avatar
Senior | Next Rank: 100 Posts
Posts: 76
Joined: Mon Jan 20, 2014 3:24 am
Thanked: 3 times

by Amadalia » Mon Apr 21, 2014 2:14 am
I didn't get your point, please elaborate more!!!!

Newbie | Next Rank: 10 Posts
Posts: 3
Joined: Sun Jun 09, 2013 1:27 am

by mayankgupta1912 » Mon Apr 21, 2014 10:56 pm
The agrgument sated can be broken down as follows:

1) More Premature Babies ------> Lesser Birth Weight
2) More Prenatal Care -----> More Birth Weight
3) More Prenatal Care -----> More Birth Weight (Conclusion)

The second and third sentences are same then how can we conclude the third sentence from the second sentence?

Please enlighten!

User avatar
Senior | Next Rank: 100 Posts
Posts: 76
Joined: Mon Jan 20, 2014 3:24 am
Thanked: 3 times

by Amadalia » Tue Apr 22, 2014 6:00 am
"How can we", isn't the point here, we can't deal with stimilus or debate it , our task is to weaken the conclusion!!!

Newbie | Next Rank: 10 Posts
Posts: 3
Joined: Sun Jun 09, 2013 1:27 am

by mayankgupta1912 » Tue Apr 22, 2014 10:49 am
Can you please tell the source of this question?

User avatar
Senior | Next Rank: 100 Posts
Posts: 76
Joined: Mon Jan 20, 2014 3:24 am
Thanked: 3 times

by Amadalia » Tue Apr 22, 2014 10:57 am
Sure, it's a question from Gmat powerscore bible
it seems it has been an official question from LSAT